Commercial Law- Mtima

अब Quizwiz के साथ अपने होमवर्क और परीक्षाओं को एस करें!

A buyer and seller agree on terms for the purchase of 100 widgets. The contract provides that the seller will ship the goods via UPS and that "risk of loss passes to the buyer on identification of the goods." The seller takes 100 widgets from its stock of 1,000 widgets and puts them in a box with a shipping label addressed to buyer. That night there is a fire in the seller's warehouse and half the widgets in the warehouse are destroyed, including all of those in the box addressed to buyer. Who bears the risk of loss of the widgets in the box? A. The buyer, because the Code provides that the risk of loss shifts to the buyer once the goods are identified as the buyer's. B. The buyer, because the contract assigned the risk to the buyer once the goods had been identified. C. The seller, because this was a shipment contract and seller bore the risk until the goods were delivered to the carrier. D. Each bears half of the risk because the fire destroyed half the goods in the warehouse.

A buyer and seller agree on terms for the purchase of 100 widgets. The contract provides that the seller will ship the goods via UPS and that "risk of loss passes to the buyer on identification of the goods." The seller takes 100 widgets from its stock of 1,000 widgets and puts them in a box with a shipping label addressed to buyer. That night there is a fire in the seller's warehouse and half the widgets in the warehouse are destroyed, including all of those in the box addressed to buyer. Who bears the risk of loss of the widgets in the box? A. The buyer, because the Code provides that the risk of loss shifts to the buyer once the goods are identified as the buyer's. B. The buyer, because the contract assigned the risk to the buyer once the goods had been identified. C. The seller, because this was a shipment contract and seller bore the risk until the goods were delivered to the carrier. D. Each bears half of the risk because the fire destroyed half the goods in the warehouse.

A seller agrees to sell a television set to a buyer for $1,000. After the deal was completed, the buyer discovers that other sellers in the area were selling the same television set for $750. The buyer claims that he is entitled to rescind the contract because under § 2-305 the price must be reasonable. Is the buyer correct? A. Yes, because the purpose of the statute is to ensure that prices are reasonable.B. Yes, because the market price is the reasonable price and this price was above market. C. No, because the reasonable price rule only applies when nothing is said as to price.D. No, because freedom of contract allows a buyer to agree to pay an unreasonable price.

Analysis. A careful reading of the statute indicates that "the price is a reasonable price" only if "nothing is said as to price." Here, there was an agreed-upon price, so the correct response is C. Although D is a true statement, it is not responsive to the call of the question. The question asked whether the buyer was correct in claiming that the price had to be reasonable under § 2-305.

A producer of cherries offers his crop to a fruit wholesaler. The producer shows the wholesaler sample cherries from his crop and, in a written agreement, the wholesaler agrees to buy the entire cherry crop for a certain price. When the cherries arrive, most of them are less plump and less ripe than the ones the producer had shown to the wholesaler. Does the wholesaler have a claim for breach of warranty?

A. Yes, because the producer told the wholesaler the cherries would be plump and ripe. B. Yes, because the sample created an express warranty that the goods would conform to that sample. C. No, because no warranty was in writing. D. No, because the wholesaler bore the risk that the goods did not conform to the sample. Analysis. This is a good example of how express warranties may be created. When the seller shows the buyer a sample, he is in effect promising that the goods will conform to the sample. Section 2-313(1)(c) provides: (c) Any sample or model which is made part of the basis of the bargain creates an express warranty that the whole of the goods shall conform to the sample or model. The correct response is B.

A contract for the sale of goods between parties who work out of their residences provides that one party has ten days to act after receiving written notice from the other party. The seller deposits a notice in the mailbox on March 1. It arrives at the home of the buyer on March 4. The buyer, however, is away on vacation and returns on March 10. On March 12, the buyer goes through the mail and reads the notice. Under these facts, when was notice received? A. March 1. B. March 4. C. March 10. D. March 12.

According to § 1-202(e): (e) Subject to subsection (f), a person "receives" a notice or notification when: (1) it comes to that person's attention; or (2) it is duly delivered in a form reasonable under the circumstances at the place of business through which the contract was made or at another location held out by that person as the place for receipt of such communications. Under these facts, it would appear that the buyer received the notice on March 4, when it was duly delivered at the buyer's place of business. The correct response is B. Under these facts, the notice was given on March 1, and the buyer had reason to know of it on March 10 and knowledge of it on March 12. Note that § 1-202(f) realistically cuts some slack for an organization that requires time to get the notice from the office that receives mail to the office of the person who is responsible for the transaction: (f) Notice, knowledge, or a notice or notification received by an organization is effective for a particular transaction from the time it is brought to the attention of the individual conducting that transaction and, in any event, from the time it would have been brought to the individual's attention if the organization had exercised due diligence. An organization exercises due diligence if it maintains reasonable routines for communicating significant information to the person conducting the transaction and there is reasonable compliance with the routines. Due diligence does not require an individual acting for the organization to communicate information unless the communication is part of the individual's regular duties or the individual has reason to know of the transaction and that the transaction would be materially affected by the information.

A seller of bottled milk and a buyer agree that the buyer will pick up the milk at 10 P.M. after the seller's business has closed. The buyer arrives and the seller tenders the milk. The buyer complains that the seller has taken no steps to package the milk in a manner that will keep it cold for the buyer. The buyer refuses the tender. Is the seller in breach? A. Yes, because the tender was not at a reasonable hour. B. Yes, because the seller knew the milk had to be kept cold and did not properly package it. C. No, because the seller may determine the time for delivery and the packaging. D. No, because the buyer must furnish facilities reasonably suited to the receipt of the goods.

According to § 2-503(1)(a), the default rule is that tender must be at a reasonable hour, but here the parties agreed to the hour. Section 2-503(1)(b) provides that "unless otherwise agreed the buyer must furnish facilities reasonably suited to the receipt of the goods." Here, they did not otherwise agree, so it was up to the buyer to provide suitable facilities. The buyer did not do so. The correct response is D.

A car dealer sells a used car with 50,000 miles on it to a buyer. Shortly thereafter, the transmission fails. Did the seller breach the implied warranty of merchantability? A. No, because used car dealers don't give a warranty of merchantability. B. No, because the implied warranty of merchantability is not given in the sale of used goods. C. Yes, because the seller has impliedly promised that the car will function properly. D. It depends whether an automobile transmission is ordinarily fit to last more than 50,000 miles.

Analysis. A merchant seller gives an implied warranty of merchantability when it sells goods, and it does not matter that the goods are used. Therefore, responses A and B cannot be correct. However, the warranty does not promise that the goods will perform perfectly. It promises only that the goods "are fit for the ordinary purposes for which such goods are used." So we would have to determine whether the car is fit for the purposes of a vehicle that has 50,000 miles on it. The best response is D.

Does paragraph 2 effectively disclaim the implied warranty of merchantability? A. Yes, because it is conspicuous and uses the word merchantability. B. Yes, because it conspicuously states that "no other warranty . . . has been or will be made." C. No, because it does not use the term "as is." D. No, because it is unconscionable to disclaim the implied warranty of merchantability.

Analysis. According to § 2-316(2), "to exclude or modify the implied warranty of merchantability or any part of it the language must mention merchantability and in case of a writing must be conspicuous." Here, paragraph 2 uses the word merchantability. According to § 1-201(b)(10)(B), the following language is conspicuous: (B) language in the body of a record or display in larger type than the surrounding text, or in contrasting type, font, or color to the surrounding text of the same size, or set off from surrounding text of the same size by symbols or other marks that call attention to the language. Here, the language is in capital letters. There may be an issue as to whether this larger type is in contrasting type to the surrounding text. In other words, if the entire agreement were in capital letters, then this disclaimer would not stand out. I think we can infer from the fact that paragraph 1 is in lowercase that the rest of the agreement probably is. If that is the case, then this language would be conspicuous. The best response is A.

Assume that a court throws out the limited remedy in paragraph 1 because it fails of its essential purpose. Will the court also throw out the exclusion of consequential damages in paragraph 3? A. Yes, because they are all part of the same warranty package. B. No, because they are in separate paragraphs. C. No, because they are different limitations of remedy. D. It depends on the jurisdiction.

Analysis. According to § 2-719(2), "[w]here circumstances cause an exclusive remedy to fail of its essential purpose, remedy may be had as provided in this Act." As we have seen, this provision makes the repair or replace remedy exclusive. It could fail if the goods are a "lemon" and the seller is not able to make them work properly. In that event, all courts agree that other remedies are available, such as revocation of acceptance under § 2-608. However, not all courts agree that the exclusion of consequential damages is thrown out along with the exclusive repair or replace remedy. The best response is D.

A buyer orders 100 widgets for $10,000. When the widgets arrive, he discovers that one of them is broken in the box. He has found another source that will sell him 100 widgets for $9,600, so he figures he can reject the widgets, buy them from the other source, and still come out ahead. He asks you for advice. Can he rightfully reject the widgets? A. Yes, because the goods fail to conform to the contract. B. Yes, because the seller has materially breached. C. No, because the seller did not breach. D. No, because his rejection would not be in good faith.

Analysis. Although the perfect tender rule purports to allow the buyer to reject the goods if they fail "in any way" to conform to the contract, because the buyer is not acting in good faith, his rejection would probably be wrongful. The best response is D. Note that C is not a correct response because even though the breach is immaterial, it is still a breach and the seller is liable for damages.

John, a law student, is selling his car to Mary. He says, "This car will get 25 miles per gallon around town. I disclaim all express warranties." The parties then enter into a partially integrated agreement that makes no mention of warranties or warranty disclaimer. Has John given Mary a warranty that the car will get 25 miles per gallon around town? A. No, because John is not a merchant seller. B. No, because the statement is puffing and not an express warranty. C. No, because all express warranties were disclaimed. D. Yes.

Analysis. An express warranty can be made by any seller, so response A is not correct. This statement is an affirmation of fact rather than puffing, because it can be objectively measured and a remedy for breach can be fashioned, so response B is not correct. The warranty has been both made and disclaimed. It is not possible to construe these statements as consistent with each other. Therefore, the warranty is given effect to protect the buyer. The correct response is D.

The buyer of a used car from a dealer signs a contract containing a merger clause that states that the writing constitutes the final expression of the parties. The agreement contains an effective disclaimer of "all warranties, express or implied, including the implied warranty of merchantability." The buyer claims that during negotiations, a salesman for the seller gave her an oral warranty against latent mechanical defects. If there were such a promise, is it excluded from the contract? A. No, because an express warranty that is both made and disclaimed is given effect. B. No, because it is part of the basis of the bargain. C. Yes, because there is an effective disclaimer of express warranties. D. Yes, because of the parol evidence rule.

Analysis. For a seller to give an express warranty, it has to make an affirmation of fact or promise that is part of the basis of the bargain. Here, the promise does not make it into the contract, not because of the disclaimer of warranties, but because of the parol evidence rule in § 2-202. This is exactly the kind of statement that the parol evidence rule is designed to exclude. The burden was on the buyer to ask that the statement be written into the contract before the contract was signed. Had it been written into the contract, it would have been given effect in spite of the disclaimer. The correct response is D.

A buyer sent an order form to a seller, offering to buy 100 widgets for $10 each, with delivery on April 1 and payment 30 days after delivery. The seller sent back an acknowledgment form to the buyer, stating that it is selling ten widgets for $100 each, with delivery on April 1 and payment 30 days after delivery. Shortly after that, the seller shipped the 10 widgets to the buyer, who promptly put them to use. The buyer's payment department then discovered the problem and asked for your advice about how much it has to pay. How would you best analyze the problem? The forms constitute an offer and an acceptance, and therefore there is a contract. The different terms would be analyzed under the rules of the jurisdiction to determine which terms are part of the contract. The forms constitute an offer and an acceptance, and therefore there is a contract, and the additional terms would be analyzed under § 2-207(2) to determine which terms are part of the contract. The forms do not form a contract under the common law mirror-image rule. The seller made a counteroffer, which the buyer accepted by conduct, so there is a contract on the seller's terms. The writings of the parties do not constitute a contract, but there is a contract by conduct under § 2- 207(3). The terms are the terms the forms did agree on, and where they did not agree, the UCC supplies the terms, such as a reasonable price under § 2-305 for the 10 widgets the buyer accepted.

Analysis. I don't think it makes any sense to analyze this problem as involving different terms under § 2-207(1). Even though the forms contain different terms, they contain different essential or "dickered" terms, and I think § 2-207 was designed to deal only with conflicting terms in the boilerplate. There is support for this position in the language of § 2-207(1) that says that "a definite . . . expression of acceptance . . . operates as an acceptance." Because the seller did not give a definite expression of acceptance, I would reject both responses A and B. While it is true that the exchange of forms did not constitute a contract under the common law, we should not use the common law if there is an applicable statute. Because § 2-207(3) speaks to this situation, I would look to it rather than to the common law, so C is incorrect. This is a situation where the writings do not establish a contract under § 2-207(1). However, conduct under § 2-207(3) establishes a contract for ten widgets because the seller shipped those goods and the buyer accepted them. The writings did not agree on price, but when the parties do not agree on price, § 2-305 states that the price is a reasonable price. Therefore, while there may be some debate, I think D is the best response.

In a contract that otherwise uses nonitalicized text, is the following disclaimer conspicuous? 3. The parties agree that the implied warranties of merchantability and fitness for a particular purpose and all other warranties, express or implied, are excluded from this transaction and shall not apply to the goods sold. A. Yes, because the italics contrast with the surrounding text. B. Yes, because a reasonable person should notice it. C. No, because every word is in the same type. D. No, because it is not larger than the other type.

Analysis. I don't think this question is easily answered. The statute requires that "a reasonable person against which it is to operate ought to have noticed it." The statute then goes on to say that it is conspicuous if in "contrasting type." There is no requirement that the type has to be larger or that every word can't be emphasized. So it would appear that italics satisfy the requirements and the best response is B. However, a court that dealt with the issue disagreed. It said, "It appears to be a classic case of attempting barely to comply with the letter of the law while circumventing its spirit. [The disclaimer] is not bolder, larger or different color type. It is not . . . indented, underscored or highlighted in any manner. It is not preceded by a distinctive legend or statement signifying the importance of that particular passage." The court's language suggests ways that the disclaimer should be made more conspicuous. The prudent drafter would take steps to do so.

A drug dealer tenders a buyer's weekly supply of cocaine and the buyer tenders a check. The dealer laughs and says that the buyer knows that it is a cash-only business. The buyer asks for more time to obtain cash and the dealer refuses. Assuming that this were a legal transaction, would the dealer have to give the buyer an extension of time to procure the cash? A. Yes, because the buyer was taken by surprise by the demand for cash. B. Yes, because it would not prejudice the seller to provide additional time. C. No, because in the ordinary course of this business only cash is accepted. D. No, because delivery is conditional on payment.

Analysis. I hope you are not offended by this question. I wanted to use an example in which everyone knows the business operates only on a cash basis (hopefully because they saw it in the movies). According to § 2-511(2), "tender of payment is sufficient when made by any means or in any manner current in the ordinary course of business." In this cash-only business, the tender was not sufficient and no additional time need be given. The best response is C. Response D is true but is not responsive to the call of the question, because the fact that delivery is conditional on payment does not explain why the buyer should not be given more time.

Both Brenda and Sarah are law students. Brenda tells Sarah that she needs word processing software to write her law review article. Sarah says, "I just bought new software, so I'll sell you my old software." Brenda buys the software and discovers that although it otherwise works perfectly well, it does not create footnotes and is therefore not useful for writing her law review article. Assuming that Article 2 applies to this transaction, has Sarah breached the implied warranty of fitness for a particular purpose? A. Yes, because she impliedly promised that the software would create footnotes. B. Yes, because the software is not fit for the ordinary purpose of word processing. C. No, because neither party is a merchant. D. No, because no warranty was given in writing.

Analysis. No warranty of merchantability is given here, because that warranty is only given by a goods merchant; and even if it were given, it was not breached because the software is fit for ordinary purposes. The elements of an implied warranty of fitness are satisfied here because the seller knew the buyer's purposes and directed her to particular goods that would satisfy that purpose. There is no requirement that the seller be a merchant or that the warranty be in writing; in fact, the warranty of fitness for a particular purpose almost always arises from the circumstances. The correct response is A.

A manufacturer of a power saw sold to consumer buyers provides a warranty that contains the following provision: Consequential Damages. In the event of a breach or repudiation of this contract by Seller, Buyer shall not be entitled to any consequential damages in excess of $1,000. The saw malfunctions and a purchaser is injured. Is the injured party likely to recover consequential damages in excess of $1,000? A. Yes, because the limitation of consequential damages will be thrown out when the limited remedy fails of its essential purpose. B. Yes, because the limitation of consequential damages is prima facie unconscionable. C. No, because parties have freedom to contract around the default rules of the Code. D. No, because the presumption of unconscionability can be rebutted.

Analysis. Response A is not relevant because there is no fact indicating that a limited remedy fails of its essential purpose. Under § 2-719(3), "[l]imitation of consequential damages for injury to the person in the case of consumer goods is prima facie unconscionable." Although the presumption can be rebutted, there are no facts here to indicate that it should be. Parties can generally contract around the default rules, but this one is clearly a regulatory rule that cannot be changed by the parties. The best response is B.

A consumer goes to a jewelry store and points out to the clerk a particular watch on display that he wishes to purchase. The clerk goes to the back of the store and emerges with a watch that he briefly shows to the consumer before wrapping it. The consumer buys the watch. When he gets home, the consumer discovers that the watch he purchased differs from the one that he pointed out in the display. Does the consumer have a claim for breach of warranty? A. Yes, because an express warranty was created by affirmation of fact or promise. B. Yes, because an express warranty was created by sample or model. C. No, because he had an opportunity to inspect the goods before purchase. D. No, because there was no language of warranty.

Analysis. Response D is not correct. Section 2-313(2) states in part that "[i]t is not necessary to the creation of an express warranty that the seller use formal words such as 'warrant' or 'guarantee' or that he have a specific intention to make a warranty." As we will see in Chapter 10, there is some support for a disclaimer of implied warranty under § 2-316(3)(b) when the buyer has an opportunity to inspect the goods for defects. This provision does not specifically apply to express warranties. An express warranty probably requires a stronger fact situation to indicate that the parties' conduct constituted a disclaimer of the warranty, so response C is not correct. Under these facts, there was no affirmation of fact or promise by the seller, so response A is not correct. However, under § 2-313(1)(c): (c) Any sample or model which is made part of the basis of the bargain creates an express warranty that the whole of the goods shall conform to the sample or model. Here, the conduct of the buyer in indicating the model of watch that he desired and the conduct of the seller in purporting to find one that conformed to that model created an express warranty that the watch sold was the same model as the watch displayed. The correct response is B.

A buyer purchases a Gizmo GPS system from a retail store. After the purchase, a third party wins a patent infringement suit against Gizmo and, because of the ensuing injunction, the system ceases to function. Does the buyer have a warranty claim against the retail store? A. Yes, because as a matter of law, the seller gave a warranty that the goods were free of infringement claims. B. Yes, because the seller gave an express warranty that the goods were free of infringement claims. C. No, because the claim for breach of warranty must be brought against Gizmo, the party who infringed, and not against the seller. D. No, because unless the buyer gets a warranty in writing, there is no warranty that the goods are free of infringement claims.

Analysis. The Code does not describe the § 2-312(3) warranty as an implied warranty; nevertheless it comes automatically with the sale. Note also that under the Code, warranties are given by the seller, not by the manufacturer. In this case it was the retail store that gave the buyer a warranty that the goods were free of infringement claims, and it gave that warranty as a matter of law rather than expressly. The correct response is A. Under the Code scheme, if a buyer brought a claim against the seller, the seller would implead the manufacturer, who would ultimately be responsible. See § 2-607(3)-(6).

A seller of consumer goods conspicuously states on the goods, "These goods are sold 'AS IS.' There are no warranties, express or implied, INCLUDING THE IMPLIED WARRANTY OF MERCHANTABILITY." Is this a legal warranty under Magnuson-Moss? A. Yes, because Magnuson-Moss does not require sellers to give warranties. B. No, because a Magnuson-Moss warranty cannot use the term "as is." C. No, because it disclaims the implied warranty of merchantability. D. No, because a Magnuson-Moss warranty must promise that the goods will be defect-free for a certain period of time.

Analysis. The correct response is A. Magnuson-Moss does not require sellers to give warranties but only to disclose the warranty terms clearly.

A farmer ordered a complex irrigation pump system from a seller. There was no disclaimer of the implied warranty of merchantability. When the pump arrived, the farmer installed it and found that it did not work properly. The farmer continued to try to get the pump to work and notified the seller of his efforts. Finally, after the farmer had the pump for two weeks, he returned it to the seller. The seller sued the farmer, claiming the attempted rejection was wrongful. Did the farmer wrongfully reject the pump? A. Yes, because he had accepted it by keeping it for two weeks. B. Yes, because the goods did not fail to conform to the contract. C. No, because he had a reasonable period of time to inspect the goods. D. No, because there was a material breach by the seller.

Analysis. The facts of this question are based on Steinmetz v. Robertus, 637 P.2d 31 (Mont. 1981). The court held that the buyer's use of the pump was not an acceptance because he was still inspecting it, as he was entitled to under the Code. The best response is C. Response B is not correct because the goods did have a warranty of merchantability, so it was promised to be a working pump. Response D is not correct because under the perfect tender rule there is no requirement of a material breach for the buyer to reject the goods.

A seller's form states on the front page, "THERE ARE NO WARRANTIES, EXPRESS OR IMPLIED." Is that language sufficient to disclaim the implied warranty of fitness for a particular purpose? A. No, because it does not use the words fitness for a particular purpose. B. No, because it is not conspicuous. C. No, because it is disclaimed only when the implied warranty of merchantability is disclaimed. D. Yes, because the disclaimer is in writing and conspicuous.

Analysis. The only statutory requirements for disclaimer of the implied warranty of fitness for a particular purpose is that the language of exclusion is in writing and conspicuous. Even though it does not disclaim the implied warranty of merchantability because it fails to use the word merchantability, this disclaimer seems to satisfy the statutory requirements for disclaimer of the implied warranty of fitness for a particular purpose. The correct response is D.

The warranty of a used car sold by a dealer states in full: "This vehicle is warranted to be free of defects for 2,000 miles or two months, whichever occurs first." A defect manifests itself at 2,500 miles. Does the purchaser have a claim for breach of warranty? A. No, because the express warranty expired after 2,000 miles. B. No, because there is no implied warranty of merchantability. C. Yes, under the express warranty. D. Yes, under the implied warranty of merchantability.

Analysis. The problem here is that although the seller gave an express warranty, the seller did not effectively disclaim the implied warranty of merchantability. Section 2-317(c) provides that "[e]xpress warranties displace inconsistent implied warranties." The issue then becomes whether in this case the express warranty displaced the implied warranty. Most courts have held that the express warranty is not inconsistent with the implied warranty and therefore does not displace it. The result would be the same if the seller attempted a disclaimer but the disclaimer was not effective. Therefore, in addition to the express warranty, the buyer got an implied warranty of merchantability and is free to argue that the car was not merchantable. The best response is D.

In a jurisdiction that applies Article 2 to the sale of electricity, an electric co-op in a small city agreed to buy "all the electricity it requires" from a supplier at a fixed price. When the market price of electricity went up in a nearby state, the co-op ordered enough electricity at the contract price so that it could satisfy its customers' needs and ship surplus electricity to that state. Is a court likely to find that the co-op was in breach of its agreement? A. No, because freedom of contract is an important principle in the Code. B. No, because no limit on requirements was stated in the contract. C. Yes, because it could only order the same amount it had ordered in previous years. D. Yes, because a reasonable measure of its requirements was the needs of its customers.

Analysis. The requirements of the buyer would probably be objectively measured by the amount needed to satisfy its customers. This is not exactly the same thing stated in response C. Although prior purchases can represent an objective measure of requirements, in this case the needs of the customers is probably a better measure. It appears here that the co-op's purchase of additional electricity was not driven by the needs of customers but was an attempt to take advantage of the market, which was not a good faith variation from its prior requirements. The best response is D.

A buyer is considering buying a car from a friend. The seller shows the buyer the title to the car, which states on its face that First Bank has a lien on the title. The parties agree on a price of $10,000 for the car, and the seller signs the title over to the buyer. Shortly thereafter, First Bank repossesses the car from the buyer because the seller was in default on his loan. Does the buyer have a claim against the seller? A. Yes, because the seller gave an implied warranty that the goods were free of liens. B. Yes, because the seller gave an implied warranty that the goods were free of infringement claims. C. No, because the implied warranty of good title is given only by merchant sellers. D. No, because the buyer had knowledge of the lien.

Analysis. The warranty of good title is given by all sellers, not just merchants. Therefore, response C is incorrect. However, § 2-312(1)(a) provides that the seller warrants that the goods are free from liens "of which the buyer at the time of contracting has no knowledge." Here, the buyer had actual knowledge of the lien because it appeared on the face of the title. In that situation, presumably the parties adjust the purchase price to account for the lien. The correct response is D.

A consumer buys a candy bar from a grocery store, bites into it, and breaks a tooth. Does the consumer have a claim for breach of warranty against the store? A. Yes, because a candy bar that breaks a tooth is probably not merchantable. B. Yes, because Magnuson-Moss provides a warranty of this consumer product. C. No, because if anyone is to blame, it is the manufacturer and not the seller. D. No, because no promises were made about the candy bar.

Analysis. There is no Magnuson-Moss claim because there is no "written warranty" under that statute. However, under § 2-314, the seller who is a goods merchant warrants that the goods are merchantable. Therefore, it appears that the buyer has a claim if he can show that a merchantable candy bar does not break teeth. The best response is A. It may indeed be the manufacturer who is to blame, but that does not bar the buyer's claim against the store since implied warranties are given by sellers. The store as a buyer of the candy bar would also have a claim against its seller for breach of warranty, so eventually the trail would lead to the manufacturer as the responsible party. We often think of this kind of case as a negligence claim, but there are many advantages to bringing it as a breach of warranty claim. For one, the claim is against the seller, which might prove more convenient. For another, it is a matter of strict liability; the plaintiff does not have to prove negligence. On the other hand, although the plaintiff can claim direct and consequential damages for breach of warranty, punitive damages and damages for "pain and suffering" cannot be recovered.

A sales receipt given to a merchant states, "All goods sold AS IS." Are the implied warranties effectively disclaimed? A. Yes, because all implied warranties are excluded by expressions like "as is." B. Yes, because the language of a specific disclaimer has been used. C. No, because in the circumstances a person would not expect to find a disclaimer on the sales receipt. D. No, the implied warranty of merchantability is not disclaimed because the word merchantability is not used.

Analysis. There is no language of specific disclaimer under § 2-316(2) here, but the words "as is" are effective language of general disclaimer under § 2-316(3) and the words are conspicuous. That provision, however, is prefaced by the ominous words, "unless the circumstances indicate otherwise." A case could be made that a reasonable person would not expect to find the terms of a contract on a sales receipt. However, this might be a better argument in the case of a consumer than in the case of a merchant. Although the issue is somewhat in doubt, I think the best response is A.

Consider the following situations: A seller of a machine tool writes on a statement to a manufacturer who buys the tool: "We will repair any defects that show up in the next 30 days." A seller sells a microwave oven that has a one-year warranty to Cisco Systems for use in the employee cafeteria. In both of these cases, the seller has given a UCC warranty. Has it given a Magnuson-Moss warranty? A. Yes, for the machine tool; yes, for the microwave oven. B. Yes, for the machine tool; no, for the microwave oven. C. No, for the machine tool; yes, for the microwave oven. D. No, for the machine tool; no, for the microwave oven.

Analysis. This is a bit tricky, as it requires careful reading of Magnuson-Moss. The machine tool does not have a Magnuson-Moss warranty, for the transaction does not involve a consumer product. The microwave oven does have a Magnuson-Moss warranty; although Cisco Systems is not a consumer, the microwave is a consumer product, and it is "consumer products" that are warranted under Magnuson-Moss. The correct response is C.

A seller of electrical supplies wrote on stationery that had his letterhead at the top, "I am offering 100-watt Luminex bulbs for 10 cents each while my supplies last." Without signing the letter, he sent it to his best customers. Is this a § 2-205 firm offer? A. Yes, but it will remain open for no more than three months. B. Yes, and it will remain open until all the lightbulbs are sold. C. No, because it is not made in a signed writing. D. No, because it does not state that it will remain open for a time stated.

Analysis. This is clearly an offer by a merchant to sell goods. Is it made in a signed writing? The definition of writing in § 1-201(b)(46) provides that " '[w]riting' includes printing, typewriting, or any other intentional reduction to tangible form." It would seem that the seller's printed stationery would constitute a writing. And the definition of signed in § 1-201(b)(37) states that it "includes any symbol executed or adopted with present intention to adopt or "accept a writing." It would seem that the seller's letterhead was adopted with the intention to indicate that the offer came from him. The Official Comment to that section elaborates: The symbol may be printed, stamped or written; it may be by initials or by thumbprint. It may be on any part of the document and in appropriate cases may be found in a billhead or letterhead. No catalog of possible situations can be complete and the court must use common sense and commercial experience in passing upon these matters. The question always is whether the symbol was executed or adopted by the party with present intention to adopt or accept the writing. Do its terms give assurance that it will be held open? I think that can be implied by the statement that the offer is open "while my supplies last." There is no time stated, but the Code would supply a reasonable time, and the Code also says that the time cannot exceed three months. Therefore, the best response is A.

Which numbered paragraph serves each function described below? A. Exclusive remedy B. Disclaimer of implied warranties C. Express warranty D. Exclusion of consequential damages

Analysis. This warranty begins in paragraph 1 by giving the buyer an express warranty under § 2-313. This is a typical "repair or replace" warranty, promising to repair or replace any defective parts for a period of time, here one year. So C=1. Note that because § 2-317 provides that warranties are cumulative, the mere fact that the seller has given an express warranty does not mean that it does not also give an implied warranty. Therefore, the seller here disclaims all warranties in the second paragraph, so B=2. The default rule of § 2-715 is that a buyer is entitled to consequential damages for breach of warranty, so the seller uses the power under § 2-719 to exclude consequential damages. Remember that according to § 2-719(3), "[l]imitation of consequential damages for injury to the person in the case of consumer goods is prima facie unconscionable." So D=3. Finally, according to § 2-719(1)(b), "resort to a remedy as provided is optional unless the remedy is expressly agreed to be exclusive, in which case it is the sole remedy." Here, the fourth paragraph constitutes the parties' agreement that the remedy is exclusive, so A=4.

A man purchased a package of frozen chicken from a grocery store. The store had placed a sticker on the package that stated "No GMOs." The man had come to the conclusion that genetically modified organisms (GMOs) are not harmful, so the designation made no difference to him. When he returned to the grocery store the next day, a sign stated that the "No GMOs" sticker had been inadvertently placed on the chicken and it in fact contained GMOs. Did the store breach an express warranty in its contract with the man? A. No, because merely stating "No GMOs" would not create an express warranty. B. No, because the man did not rely on the statement in making his purchase. C. Yes, because the seller made an affirmation of fact which relates to the goods that turned out not to betrue. D. It depends on the jurisdiction.

Analysis. Under the facts, there was an affirmation of fact made by the seller to the buyer which related to the goods, so response C seems to be correct. However, to create an express warranty, the affirmation of fact or promise must become "part of the basis of the bargain." It can be argued that this affirmation of fact did not become part of the basis of the bargain because the man did not rely on it, so response B may also be correct. Because this question can go either way, the best response is D. We will revisit the issue of "basis of the bargain" when we look at express warranties made by remote sellers in Chapter 11.

Statute reader. In which of the following situations is the buyer not entitled to revoke acceptance under § 2-608? A. The seller tenders a complex machine that looks fine, but after the buyer sets it up, the machine does not work at all. B. The seller tenders a complex machine. The buyer notices some defects in it on delivery, but the seller assures the buyer that she will repair the problems. The problems turn out to be serious and the seller does not fix them. C. The seller tenders a complex machine and, before the buyer has a chance to accept it, the seller assures the buyer it is in good condition. It turns out to have serious problems. D. The seller tenders a complex machine that looks fine, but after the buyer sets it up, the machine has a problem with a part that needs to be replaced and that the seller has in stock.

Analysis. Under § 2-608, the buyer may have a remedy if there is a "nonconformity [that] substantially impairs its value to him." This means that the goods do not conform to what was promised. Notice, however, that there are additional requirements. The buyer may revoke only if the goods were accepted: (a) on the reasonable assumption that its non-conformity would be cured and it has not been seasonably cured; or (b) without discovery of such non-conformity if his acceptance was reasonably induced either by the difficulty of discovery before acceptance or by the seller's assurances. Response A sounds like the § 2-608(1)(b) situation where there is "difficulty of discovery before acceptance." Response B sounds like the § 2-608(1)(a) situation where the buyer reasonably assumed the nonconformity would be cured. Response C sounds like the § 2-608(1)(b) situation where the acceptance was induced by the seller's assurances. In response D, it does not sound like the nonconformity "substantially impairs the value" of the machine to the buyer. In that case, the § 2-608 remedy is not available. Therefore, D is the correct response.

A buyer orders a widget for $100. When she goes to pick it up, she tells the seller, "I am so lucky to get that widget. It is the last one available in town and I need it to keep my machine going." The seller's eyes light up and he says, "The price just went up to $200." The buyer reluctantly agrees to pay, but then sues to get the additional $100 back. Is the modification enforceable? A. Yes, because the modification does not require consideration. B. Yes, because the buyer was free not to agree to it. C. No, because there was no consideration for the promise to pay more. D. No, because the seller did not act in good faith.

Article 2 does not require consideration to support a modification, but that does not mean that all modifications entered into without consideration are enforceable. As stated in § 1-304, the obligation of good faith is found in every contract. The request for a modification can sometimes come from a chiseler, as it did here. The best response is D. Official Comment 2 to § 2-209 states: However, modifications made [under subsection (1)] must meet the test of good faith imposed by this Act. The effective use of bad faith to escape performance on the original contract terms is barred, and the extortion of a "modification" without legitimate commercial reason is ineffective as a violation of the duty of good faith.

Mary owns a work of art that is worth $20,000. John purchases the work for $19,000, giving Mary a check that bounces. Before Mary can do anything about it, John has sold the work out of the back of his truck to Arthur for $4,000. Does Mary have a claim against Arthur? A. Yes, because Arthur is not a good faith purchaser for value. B. Yes, because Arthur got only void title from John. C. No, because Arthur is a good faith purchaser for value. D. No, because John got good title and transferred good title to Arthur.

Because he paid with a bad check, John got only voidable title under § 2-403(1)(b). Therefore, responses B and D cannot be correct. The rule of § 2- 403(1) is that "[a] person with voidable title has power to transfer a good title to a good faith purchaser for value." So the issue comes down to whether Arthur was a good faith purchaser for value. He was clearly a purchaser because he took by sale. And he purchased for value, as the law does not inquire into the adequacy of consideration, so $4,000 can be consideration for a $20,000 painting. However, it seems to me that a person does not act in good faith when he buys valuable goods off the back of a truck for a lot less than they are worth. In those circumstances, a reasonable purchaser would make further inquiry into the source of the goods. So I think the best response is A. See Hollis v. Chamberlin, 243 Ark. 201 (1967). Extra credit: If Arthur loses to Mary, does Arthur have a claim against John? Sure. When John sold the goods to Arthur, he gave Arthur a warranty of title under § 2-312(1), so Arthur would have a claim against John for breach of that Warranty.

A seller agrees to deliver 1,000 widgets by June 1. The widgets are not tendered until June 8. The buyer accepts delivery of the widgets and then informs the seller that it is seeking damages for the late delivery. Assuming the buyer suffered some loss, is the buyer entitled to damages? A. Yes, because the seller did not tender the widgets in conformity with the contract. B. No, because the breach was immaterial. C. No, because time is not of the essence. D. No, because the buyer waived its rights by accepting the late delivery.

Because of the nonconformity in tender, the buyer might have had the right to reject the widgets under § 2-601. Because it accepted the goods, however, it is too late to reject. But acceptance does not waive the right to recover damages for breach. See § 2-607(2). The correct response is A.

A seller promises delivery of 100 type-X widgets by June 1. From past dealings, the seller knows that the buyer keeps a stock of the widgets on hand to replace worn-out parts on its assembly line, and a typical supply lasts three months. On June 1, the seller delivers only 70 type-X widgets. The seller promises that it will have 30 more in three weeks and asks for the additional time, promising to pay for any losses the buyer incurs because of the delay. The buyer asks your advice. Does the buyer have to give the seller additional time? A. Yes, because the time for performance has not yet expired. B. Yes, because the seller would reasonably believe the offer of late tender would be acceptable. C. No, because a seller is never entitled to cure after the time for performance has expired. D. No, because the seller's breach was material.

Because the time for performance expired on June 1, this question falls under § 2-508(2) and not § 2-508(1). Because of the past experience, it would seem that the seller had reasonable grounds to believe the nonconforming tender would be acceptable. It all turns on whether the additional three weeks the seller seeks is "a further reasonable time." It would appear that because the seller knows that the stock lasts for three months, the seller had reasonable grounds to believe that a three-week delay would be reasonable. The best response is B. Note that the analysis would be simpler under Amended § 2-508(2), for it would not turn on what the seller had reasonable grounds to believe.

Barney, a law student, buys a used car from Bitterroot Motors, a car dealer. The dealer signs over the title and gives Barney the keys. Barney asks them if it would be all right if he picked up the car the next day and they tell him it is not a problem. That night, a tree limb breaks off and damages the car, which is parked at the dealership. Who is responsible for the loss? A. Bitterroot Motors, because Barney did not have receipt of the car. B. Bitterroot Motors, because it had not tendered the car. C. Barney, because Bitterroot Motors had tendered the car. D. Barney, because he had receipt of the car.

Bitterroot Motors is a merchant. Therefore, according to § 2-509(3), "the risk of loss passes to the buyer on his receipt of the goods." Receipt means physical possession. Even though Barney may have had constructive receipt because he had the title and the keys, he did not have physical possession. Therefore, the correct response is A. This seems to make sense as a matter of policy, because the merchant is in a better position to care for and insure the goods while they are under its control.

Assume in the Frigaliment case that the buyer offered evidence that in the poultry business, chicken meant broilers, and the seller offered evidence that in a previous contract between the parties, the buyer accepted stewing chicken without objection. Which meaning would govern? A. Broilers, because trade usage prevails over course of performance. B. Broilers, because trade usage prevails over course of dealing. C. Stewers, because course of performance prevails over trade usage. D. Stewers, because course of dealing prevails over trade usage.

First, let's characterize the evidence. The buyer's evidence of meaning in the poultry business is trade usage; the seller's evidence from a previous contract between the parties is course of dealing. According to § 1-303(e)(3), "course of dealing prevails over usage of trade." Therefore, response D is correct.

A law student walks into a bookstore and says, "I'm here to pick up that copy of The Glannon Guide to Sales I ordered." The clerk hands her The Glannon Guide to Secured Transactions. Has there been tender of delivery? A. No, because this is not an Article 2 transaction as the law student is not a merchant. B. No, because the goods are nonconforming. C. No, because the time and place for tender have not been determined. D. Yes, there has been a tender and there has also been a breach.

I hope you are still not being fooled by response A. If you are, please review Chapter 4.B. Here, the goods have been tendered at a reasonable time and place under § 2-503, so C is not a correct response. The reason tender of delivery is so important is because, according to § 2-507, once there has been tender and notice, then payment is due. But § 2-503(1) provides that "[t]ender of delivery requires that the seller put and hold conforming goods at the buyer's disposition." If the tender is of nonconforming goods, the buyer may prefer not to accept delivery rather than have a claim for breach. Here, there was not a tender of conforming goods, so the correct response is B.

A buyer orders ten widgets for future delivery. In between the time the contract is made and the time for performance, the cost of widget-making increases substantially. The seller asks the buyer if it would agree to pay an additional 5% to reduce the seller's loss. The buyer refuses. The seller delivers, but then claims that the buyer is in breach of the duty of good faith for failing to agree to the modification. Is the buyer in breach? A. No, because there is no requirement that a party agree to a modification. B. No, because the Code does not require a party to act in good faith. C. Yes, because the buyer took advantage of the seller's situation. D. Yes, because the request for a modification was fair and equitable in view of the circumstances.

I included this question to make sure you don't take the notion of good faith too far. The fact that there is a reasonable request for a modification does not require the other party to agree to it. If you "just say no," that is not lack of good faith. It is simply insisting on the benefit of the bargain you made. The correct response is A.

A distributor of chickens orders frying chickens from a chicken processor. The chicken is to be delivered in five installments. For the first installment, the processor tenders stewing chicken. The distributor accepts the delivery under protest, for it feels it must provide chicken to its customers, even if it is not the right kind of chicken. The distributor finds, however, that its customers are unhappy with the chicken and refuse to buy any more from the distributor. The processor is willing and able to tender frying chicken for the next four installments. Can the distributor cancel the entire contract? A. No, because the nonconformity in the first installment did not substantially impair the value of that installment. B. No, because the nonconformity in the first installment did not substantially impair the value of the whole contract. C. Yes, because the nonconformity in the first installment substantially impaired the value of that installment. D. Yes, because the nonconformity in the first installment substantially impaired the value of the whole contract.

I think these facts provide a good example of when the nonconformity in the first installment substantially impairs the value of the whole contract. Even though the seller is willing to make conforming deliveries in the future, those deliveries will be of no value to the distributor, because it has lost its customers. The best response is D.

In a contract for the sale of goods, a merchant offeror's form states, "This contract is governed by the law of Texas." The merchant offeree's form states, "This contract is governed by the law of Vermont." In a jurisdiction that treats the offeror as having objected to different terms, which jurisdiction supplies the governing law? A. Texas, because the offeror should be able to choose the applicable law. B. Texas, because Vermont is materially different. C. Vermont, because it is not materially different. D. Whichever state prevails when choice of law rules are applied.

In this hypothetical, the offeree's form contains a different term. The Code in § 2-207(2) does not tell us what to do with different terms. In a jurisdiction that treats the offeror as having objected to different terms, the offeror's term would govern. The correct response is A. Note that the correct response is not B, because we are not determining the governing term under § 2- 207(2)(b). Under this approach, the offeror gets its term whether the offeree's term is materially different or not.

A buyer and a seller entered into a fully integrated contract for the sale of "1,000 red pens." The seller tendered pens that had a silver body but wrote with red ink. The buyer claimed that the parties intended that the contract called for pens with a red body. Which of the following facts should be determinative of the outcome in the case? A. The seller had in mind pens that wrote with red ink when the contract was signed. B. In the pen business, the color generally describes the color of the ink. C. The contract was fully integrated. D. During pre-contract negotiations, the seller had sent the buyer a color photograph of the pen.

Interpretation issues arise when language is ambiguous, meaning it is susceptible to more than one meaning. Here the language can reasonably be interpreted either way. Even though the contract was fully integrated, it makes no sense to refuse to admit evidence for the purpose of resolving an ambiguity in language, so C is not a correct response. Response A describes subjective evidence. It is not reliable, and it does not indicate that the parties agreed on that meaning. Response B describes trade usage. It is reliable, and in the absence of agreement by the parties, would probably be the best response. However, when there is objective evidence that indicates what the parties have intended the language to mean, that evidence should be given greater weight. The best response is D.

A nonmerchant buyer talks with a nonmerchant seller on the phone, and agrees to buy the seller's Frank Lloyd Wright chair for $60,000. The seller then changes her mind and refuses to sell. The buyer sues the seller. When the buyer takes the seller's deposition, the seller testifies that she did in fact agree to sell the chair for $60,000, but her lawyer told her that there was a Statute of Frauds for her protection. It gave her a chance to reflect on the transaction before writing anything down, and she had changed her mind before committing the agreement to writing. Is the agreement enforceable? No, because there is no writing signed by the seller, the party against whom enforcement is sought. No, because one of the purposes of the Statute of Frauds is the cautionary function, that makes peopleconsider serious transactions. Yes, because she admitted it in her deposition. Yes, because oral contracts between nonmerchants are enforceable even if the amount is more than$500.

It is often said that one of the purposes of the Statute of Frauds is its cautionary function. However, the drafters of the Code apparently thought that it would do little harm to enforce agreements that the parties actually made. They therefore created the admission exception in § 2-201(3)(c). Here, the deposition satisfies that exception, for a deposition is testimony under oath. The correct response is C. For Judge Richard Posner's take on whether a party who claims the benefit of the statute can be compelled to submit to a deposition, see DF Activities Corp. v. Brown, 851 F.2d 920 (7th Cir. 1988). Judge Posner said, "if defendant swears in an affidavit that there was no contract, we see no point in keeping the lawsuit alive; although defendant may blurt out an admission in a deposition, . . . this is hardly likely."

A commercial buyer receives a shipment of goods from a seller and stores them without inspecting them. Three months later, the buyer discovers a defect in the goods. The buyer immediately informs the seller that it is rejecting the goods and sends them back. Is the rejection a rightful rejection? A. Yes, because the buyer had a reasonable time to conduct an inspection. B. Yes, because the buyer could revoke its acceptance. C. No, because the buyer had accepted the goods. D. No, because the buyer has waived its right to recover damages from the seller.

It seems that a reasonable time to conduct an inspection has long passed, so A is not a correct response. Whether the buyer can revoke acceptance is not the call of the question — you are asked about rejection — so B is not a correct response. D is not a correct response partly because it is not responsive to the call of the question and partly because it is not substantively correct. A buyer does not waive its rights to recover damages by accepting nonconforming goods. The correct response is C. Even though there is no affirmative act constituting acceptance, the buyer's retention of the goods for a substantial time amounts to an acceptance under § 2-606.

Buyer and seller negotiated a detailed multi-page agreement over a period of weeks that contained a merger clause. After the agreement was signed, the seller discovered that the agreement stated, "Delivery shall begin in __ days." Seller claimed that during negotiations, the parties had talked about delivery in 60 days but the buyer claimed the discussion involved 30 days. Is evidence of the parties' negotiations admissible? A. Yes, because the writing was not intended to be a complete and final expression of their agreement as to the delivery date. B. Yes, because the issue is one of interpretation of an ambiguous term. C. No, because the agreement was intended to be complete and exclusive. D. No, because the evidence would contradict a term of the writing.

It seems to me that this agreement cannot be final on the delivery term because it is clear from the face of the agreement that the parties did not reach agreement on that term. Therefore, the written agreement is only a partial integration. It may be final and complete on everything else, but not on delivery, so evidence can be admitted on that issue. The best response is A.

Mary and John agree that John will sell Mary an autographed Ted Williams baseball for $1,000. John then e-mails a friend with a copy to Mary: "I just agreed to sell that autographed Ted Williams baseball to Mary for $1,000." Before the exchange, Mary says to John, "I admit I made that agreement with you, but I changed my mind. I am not going to buy it." Is the agreement enforceable? A. Yes, because John's e-mail to the friend evidences it. B. Yes, because John's sending a copy of the e-mail to Mary acts as a confirmation. C. Yes, because Mary admitted making it. D. No, because there is no writing signed by the party against whom enforcement is sought and no exception applies.

John's e-mail to the friend would probably be sufficient against John, but he is not the party against whom enforcement is sought. There is no writing signed by Mary. The confirmation exception does not apply because it appears that John and Mary are not merchants. The admission exception does not apply because the admission was not in a "pleading, testimony or otherwise in court." Because no exception applies, the correct response is D.

A seller sells a machine to a buyer. The contract provides that the seller will service it "regularly" for three years. For a year, the seller services it every week. Then the seller finds out that in the trade, these machines are only serviced monthly, so the seller insists that he only has to service the machine monthly. Is the seller right? A. Yes, because the trade usage is monthly. B. Yes, because the course of dealing is monthly. C. No, because the express term is weekly. D. No, because the course of performance is weekly.

Note on test taking. When you see a question that is very similar to a previous question, identify the factual differences. The assessor probably intends different facts to lead to a different conclusion. The important facts are that the express term of the agreement is "regularly," which is an undefined term, the course of performance is weekly, and the trade usage is monthly. In the Code hierarchy of interpretation, course of performance governs over trade usage. Therefore, the correct response is D.

In a contract for the sale of goods, a merchant offeror's form states, "This contract is governed by the law of Texas." The merchant offeree's form states, "This contract is governed by the law of Vermont." In a jurisdiction that treats different terms the same as additional terms, which jurisdiction supplies the governing law? A. Texas, because the offeror should be able to choose the applicable law. B. Texas, because Vermont is materially different. C. Vermont, because it is not materially different from Texas. D. Whichever state prevails when choice of law rules are applied.

Note on test taking: The responses to this question are identical to those of the previous question, so the correct response is likely to turn on a difference in facts. The difference is that the previous question asked us to apply the knockout rule, and this question asks us to treat different terms the same as additional terms. Because we are treating different terms like additional terms, § 2-207(2) tells us that the different term (Vermont law) in the offeree's form is being proposed as an addition to the contract. Because both parties are merchants, the term presumptively becomes part of the contract unless it is eliminated in one of the three ways. Subsections (a) and (c) are not satisfied by the facts, so under subsection (b) it turns on whether Vermont choice of law materially alters Texas choice of law. Initially, one might think the two terms are materially different, but note that the issue is whose law will apply — and in both states the applicable law is the UCC, which by definition is unlikely to be materially different in Vermont (although it could be argued that a particular provision at issue in the case was materially different). So if the offeree's different term does not materially alter the offeror's term, then the offeree's term, Vermont law, is in the contract, and the offeror's term, Texas law, is out. The best response is C.

John and Mary agreed on the telephone that John would buy Mary's Ted Williams autographed baseball for $400. Mary later told him, "Ha, Ha. I got a better offer for that ball, and all I had with you was a verbal agreement, so that doesn't count." Is there an enforceable contract between John and Mary? A. Yes, because oral agreements for the sale of goods for less than $500 are enforceable. B. Yes, because the UCC does not apply to this agreement because John and Mary are not merchants. C. Yes, because Mary admitted making the agreement. D. No, because there is no writing signed by Mary.

Of course, we know that Mary meant to say, "all I had with you was an oral agreement." Because this is a transaction in goods, the UCC applies. Under § 2-201(1), this agreement does not need to be evidenced by a writing because it is not "a contract for the sale of goods for the price of $500 or more." Therefore, response A is correct.

A long-term contract between a commercial buyer and a commercial seller provides that "Seller retains the right to modify the agreement on ten days' written notice to Buyer." After the agreement has been in effect for two years, the seller notified the buyer that pursuant to this provision, "the price of the goods in the next delivery will increase by 7% to reflect an increase in the cost of raw materials, and any disputes will be resolved by arbitration pursuant to the rules of the American Arbitration Association." Which of the modifications is a court likely to enforce? A. Both the price increase and the arbitration provision. B. The price increase, but not the arbitration provision. C. The arbitration provision, but not the price increase. D. Neither the price increase nor the arbitration provision.

Provisions such as this are beginning to appear in many consumer contracts. Even though a court might be more likely to enforce it in a negotiated contract, I have concerns about it. If taken literally, the provision gives the seller the power to rewrite the contract, which would make the initial agreement illusory and meaningless. On the other hand, parties should be able to plan to alter their agreement to address future situations. I think a fair compromise would be to allow the seller to make modifications that reflect an objective change in the parties' situation. For example, it is reasonable to allow the seller to reset the price based on changing circumstances. However, it seems unreasonable to allow the seller to use this power to obtain advantages that it did not initially bargain for, such as using arbitration as a forum. While the law on this matter is uncertain, I think the best response is B.

ABC Corp. and XYZ Corp. negotiated a detailed multi-page agreement over a period of weeks that did not contain a merger clause. After the agreement was signed, seller sought to admit evidence of a term that supplemented the agreement. Should the evidence be barred by the parol evidence rule? Yes, because the writing was intended to be a complete and exclusive statement of the terms of the agreement. Yes, because the evidence contradicts the writing. No, because there is no merger clause in the agreement. No, because the writing was not intended to be a final expression of their agreement.

Recall that the intention of the parties is the key factor in determining whether a writing is intended to be complete and exclusive. Often, a merger clause is evidence of that the intent, but here there is no merger clause. On the other hand, we are told that this was a detailed agreement negotiated over a period of weeks by two corporations. It seems to me that those facts and circumstances indicate the intent of the parties was to embody their agreement in the writing. The presence or absence of a merger clause may be a strong factor in ascertaining the intent, but it is not determinative. See, for example Arb, Inc. v. E-Systems, Inc., 663 F.2d 189 (D.C. Cir. 1980), in which the court stated: Here, the bulk of the evidence suggests that the parties intended the written contract to be the complete and exclusive statement of the terms of their agreement. The length of the contract, its exhaustive detail, and the prolonged period of negotiation preceding its signing, collectively considered, support this conclusion. The best response is A.

A buyer and a seller of widgets have entered into a written contract that was prepared by the buyer and is the complete and exclusive statement of the terms of their agreement. The buyer claims that a term in the agreement is ambiguous and wants to offer evidence to prove its meaning. The seller claims that the term is not ambiguous and resists the buyer's offer of evidence. What should the court do? A. Exclude the evidence because the writing is complete and exclusive. B. Determine whether the term is ambiguous. C. Admit the evidence for the purpose of determining the meaning as a question of fact. D. Interpret the term against the buyer, as the buyer was the drafter.

Response A is not correct because when we say that the writing is "complete and exclusive," we are saying that all the terms are found in the writing. However, that says nothing about the meaning of those terms. To resolve an issue of interpretation, the court will first determine as a matter of law whether the term is ambiguous. Therefore, the correct response is B. Note that I have bypassed the issue of what test will be used to determine whether the term is ambiguous because that differs from state to state. If the court determines that the term is ambiguous as a matter of law, then evidence may come in to prove its meaning as a question of fact. Response C is incorrect because it represents the second step in the process. One consideration in determining that fact question is which party drafted the agreement. Response D is incorrect because before the evidence comes in to determine what the meaning is as a question of fact, it must be determined whether the language is ambiguous as a matter of law.

John agreed to sell his car to Mary for $25,000. John then went to his lawyer's office, but his lawyer was not there, so he left a note for her that said: "I've just sold my car to Mary for $25,000. What documents do I need to complete the transaction? s/John." That night, John got a better offer for the car and told Mary the agreement was not enforceable because it was oral. Is there an enforceable agreement between John and Mary? A. No, because serious agreements for large amounts of money must always be reduced to writing. B. No, because there is no sufficient writing signed by Mary. C. Yes, because it is unethical for John to try to refuse enforcement of this contract. D. Yes, because there is a sufficient writing signed by John.

Response A suggests one of the policy reasons behind the Statute of Frauds — it is a reminder to a person that this is a serious matter and serious matters should be reduced to writing. This is sometimes called the channeling function — it channels our behavior into a more desirable pattern. Nevertheless, there is no such general rule. Similarly, Response C suggests one of the policy reasons behind enforcing oral agreements — if the purpose of the Statute is to prevent fraud, how is that purpose served when people refuse to honor agreements that they really made? Nevertheless, a person is free to raise the defense even if the person made the contract, subject to an exception we look at shortly. Because this agreement is within the Statute of Frauds, it needs to be evidenced by a writing signed by John, the party against whom enforcement is sought. The note to his attorney satisfies the requirement of a "writing sufficient to indicate that a contract for sale has been made between the parties and signed by the party against whom enforcement is sought." Therefore, D is the correct response. You might wonder as a practical matter how Mary could enforce the agreement, but if a suit were brought, the memorandum would probably be discoverable.

Analysis. Under § 2-608, the buyer may have a remedy if there is a "nonconformity [that] substantially impairs its value to him." This means that the goods do not conform to what was promised. Notice, however, that there are additional requirements. The buyer may revoke only if the goods were accepted: (a) on the reasonable assumption that its non-conformity would be cured and it has not been seasonably cured; or (b) without discovery of such non-conformity if his acceptance was reasonably induced either by the difficulty of discovery before acceptance or by the seller's assurances. Response A sounds like the § 2-608(1)(b) situation where there is "difficulty of discovery before acceptance." Response B sounds like the § 2-608(1)(a) situation where the buyer reasonably assumed the nonconformity would be cured. Response C sounds like the § 2-608(1)(b) situation where the acceptance was induced by the seller's assurances. In response D, it does not sound like the nonconformity "substantially impairs the value" of the machine to the buyer. In that case, the § 2-608 remedy is not available. Therefore, D is the correct response.

Response B does not correctly state the rule. Even if the buyer discovered the defect during the period of inspection, he is free to accept the goods and seek damages. It appears that the buyer has no grounds for revocation of acceptance, so response C is not correct. The general rule that the buyer has an obligation to pay the price of goods accepted is correctly expressed in response A. However, the buyer also has a claim for damages for breach of warranty, and according to § 2-717, the buyer may deduct damages from the price due if he notifies the seller of his intention to do so. Therefore, D is the best response.

A consumer calls a computer seller to purchase a computer. The essential terms, such as the description of the goods and the purchase price, are discussed. When the computer arrives, the consumer finds a form purporting to be a contract. The contract contains some terms that are onerous, but not unconscionable. Is the consumer bound by them? A. No. Because the contract is a confirmation, the terms are binding only if they do not materially alter the terms that would be implied by the default rules. B. Yes, because as a practical matter the seller could not discuss all the terms over the telephone, the consumer should have expected additional terms to follow. C. Yes, but as with all consumer contracts, the buyer has a three-day "cooling-off period" to decide whether to keep the goods. D. It depends on the jurisdiction. Some would go with response A and others response B.

Response C is incorrect, although it is a popular myth. Consumer protection statutes that provide a three-day "cooling-off period" generally apply only to door-to-door sales and telephone solicitation sales initiated by the seller. This is law, so "it depends on the jurisdiction" is a pretty reliable answer. Response A represents the holding in Klocek v. Gateway, Inc., 104 F. Supp. 2d 1332 (D. Kan. 2000). Response B represents the holding in Hill v. Gateway 2000, Inc., 105 F.3d 1147 (7th Cir. 1997). The correct response is D.

Pursuant to a contract, a merchant seller shipped a railroad car full of apples to a merchant buyer in Las Vegas in July. The buyer looked at them and determined that they were not the kind of apple specified in the contract. The buyer immediately informed the seller, but when the buyer did not receive any instructions from the seller, the buyer told the railroad to put them on a siding. By the time the seller got around to doing anything, the apples had spoiled. Who is responsible for the loss? A. The seller, because the risk of loss remains on a seller who ships nonconforming goods. B. The seller, because it failed to provide instructions to the buyer. C. The buyer, because it had a duty to make reasonable efforts to sell the apples for the seller's account. D. The buyer, because it had a duty to accept the apples even though they were the wrong kind.

Response D is incorrect, because according to § 2-601, a buyer has the right to reject the whole when there is a substantial nonconformity. While it is true under § 2-510 that the risk of loss remains on the seller when the seller delivers nonconforming goods, this is not the kind of loss contemplated by that section. Therefore, response A is incorrect. Because the buyer is a merchant, the rules on the buyer's duties are found in § 2-603. Response B is incorrect because the buyer has duties with respect to perishable goods even if the seller does not provide instructions to the buyer. Response C is correct because the buyer has the duty "in the absence of such instructions to make reasonable efforts to sell [the goods] for the seller's account if they are perishable or threaten to decline in value speedily."

A farmer orders a water pump that the seller promises will pump 1,000 gallons per minute. The farmer inspects the pump on arrival and finds that it will pump only 950 gallons per minute. The seller promises to make it work properly, so the farmer accepts it and pays for it. Later, it becomes clear that the seller cannot improve the pumping capacity but had hoped that the farmer would accept the slightly reduced capacity. In fact, unknown to the seller, the farmer requires the full capacity to fully irrigate his fields. Can the farmer revoke his acceptance? A. Yes, because the nonconformity substantially impairs its value to him. B. Yes, because he accepted it on the reasonable assumption that its nonconformity would be cured and it has not been seasonably cured. C. Yes, because both elements described in responses A and B are satisfied. D. No, because once there is acceptance it is too late to reject.

Response D is true but not relevant because the buyer is not attempting to reject; he is attempting to revoke his acceptance. The applicable statute is § 2-608. One element is that "the nonconformity substantially impairs its value to him." In this situation, a reasonable seller might not know that a reduced capacity of only 5% would be a substantial impairment. But the test is not what the seller knew; rather it is whether the nonconformity impaired the value to the buyer, and here it did. But the buyer has to satisfy one of two more tests, one of which is that "he has accepted it . . . on the reasonable assumption that its nonconformity would be cured and it has not been seasonably cured." Because he can satisfy both elements, the buyer can revoke acceptance. The correct response is C.

A buyer orders ten widgets for future delivery. In between the time the contract is made and the time for performance, the cost of widget-making increases substantially. The seller asks the buyer if it would agree to pay an additional 5% to reduce the seller's loss. The buyer agrees. After delivery, the buyer refuses to pay more than the original contract price, claiming the modification is not enforceable. Is it enforceable? A. No, because there was no consideration for the promise to pay more. B. No, because the agreement was not in writing. C. Yes, because the modification does not require consideration. D. Yes, because the modification was fair and equitable in view of the circumstances.

Response D states the general rule from the Restatement, but this contract is for the sale of goods, so the Article 2 rule is applicable. There is no general requirement that a modification be in writing. Under § 2-209(1), there is no requirement of consideration for a modification. There are no facts suggesting that the modification was not made in good faith. Therefore, C is the correct response.

Sarah, a law student, sells her car to Barney, another law student. Sarah signs over the title and gives Barney the keys. He tells her that he will pick it up from her house the next morning and she says it will be waiting for him. That night, a tree limb breaks off and damages the car, which is parked in Sarah's driveway. Who is responsible for the loss? A. Sarah, because Barney did not have receipt of the car. B. Sarah, because she had not tendered the car. C. Barney, because Sarah had tendered the car. D. Barney, because he had receipt of the car.

Sarah is not a merchant. Therefore, according to § 2-509(3), "the risk passes to the buyer on tender of delivery." Sarah tendered delivery when she gave the title and the keys to Barney. Because the risk passed at that moment, Barney became responsible for the loss. The correct response is C. This seems to make sense as a matter of policy, for Sarah is likely to cancel her insurance after she has sold the car.

A seller agrees to sell 10,000 bricks to a buyer for $2,000 under a contract that states that payment is due on delivery. The buyer comes to the seller's place of business to get the bricks. The seller demands payment, but the buyer refuses to pay, claiming that it is customary in the trade that a buyer has 30 days after delivery to pay. The seller refuses to give the bricks to the buyer. Who is in breach? A. The seller, because the rule is that delivery must be made before payment is made. B. The seller, because the trade usage would be read into the contract. C. The buyer, because the rule is that payment must be made before delivery is made. D. The buyer, because the trade usage is trumped by an express term of the contract.

Responses A and C do not state the rule correctly. According to §§ 2- 507 and 2-511, delivery and payment are conditional on each other, so they must be tendered simultaneously. As a general rule, according to § 1-303, usage of trade is read into the contract. However, the express terms of the contract prevail over the trade usage under § 1-303(e)(1). Here, the contract specifically states that payment is due on delivery. That express term is inconsistent with the trade usage, so it governs. The correct response is D. Let's now turn to risk of loss. When delivery is at the seller's place of business, according to § 2-509(3), "the risk of loss passes to the buyer on his receipt of the goods if the seller is a merchant; otherwise the risk passes to the buyer on tender of delivery." Recall that a tender is an offer to perform, whereas receipt of goods, according to § 2-103(1)(c), means "taking physical possession of them." So if the buyer is offered the goods, but decides to come back later to pick them up, there has been tender but not receipt. Note that which rule is applicable turns on whether the seller is a merchant.

John Smith takes his Rolex watch to be repaired by Alice's watch repair shop. After the watch has been repaired, it is inadvertently put in the display case until John can pick it up. There is a tag on it that says "John Smith $72," with the "$72" representing the cost of repair. An inexperienced clerk sells the watch, which is worth $2,000, to Martin for $72. Does Martin have good title in the watch? A. Yes, because he is a good faith purchaser for value. B. Yes, because he is a buyer in ordinary course of business. C. No, because he should have known that the sale violates the rights of another person in the goods. D. No, because he is not a buyer in ordinary course of business.

The combination of the name tag and the low price for a Rolex seem to me circumstances in which a reasonable person would not have assumed he was buying a watch out of the inventory of the seller. Although this may be a close case, I think the best response is D. If you do not agree, I hope you chose response B, because that is the standard applicable to a purchase from an entruster.

During negotiations, the seller of a boat promised the buyer that the engine was warranted to be free of defects for a period of two years. The parties eventually signed a written agreement that contained no express warranty terms. Fourteen months after the purchase, the engine developed problems because of a defect. A court found that the writing is a complete and exclusive statement of the terms of the agreement. Is the seller liable under the oral warranty? A. It remains to be determined as a question of fact whether the promise was made. B. Yes, because the oral promise does not contradict the terms of the writing. C. No, because the writing was a complete and exclusive expression of their agreement. D. No, because the buyer would have a claim even under the written warranty.

The court has determined that the agreement is a full integration. Therefore, no evidence may be offered to contradict or to supplement it. Here, even though the evidence supplements, it is excluded. If it is excluded as a matter of law, then it doesn't matter whether, as a question of fact, the agreement was actually made. The correct response is C. Some say that in cases like this the parol evidence rule elevates efficiency at the expense of truth.

Mary Jones called John Smith and ordered 1,000 red widgets at a price of $1 each. Smith wrote on a notepad that was headed "From the desk of John Smith," these notes: "8/29/19 sold red widgets $1 each Mary Jones." Smith later could not find the notes and did not fill the order. Did the parties have an enforceable contract? A. Yes, because the writing requirement was satisfied. B. No, because Smith did not sign the writing. C. No, because the writing lacked an essential term. D. No, because the writing could not be found.

The definition of "signed" in § 1-201(b)(39) provides that "'Signed' includes any symbol executed or adopted with present intention to adopt or accept a writing." The notepad contained the heading "From the desk of John Smith," which seems to indicate that it was adopted as a writing of Smith's. Therefore, B is not a correct response. Curiously, cases have held that it is sufficient that there has been a writing that evidenced the agreement even if the writing was lost. Therefore, D is not a correct response. However, the writing is lacking a quantity term, which the weight of authority holds makes the writing insufficient to evidence the agreement. Therefore, A is not a correct response and C is the correct response.

ABC Corp. and XYZ Corp. negotiated a multi-page agreement over a period of weeks that contained a merger clause. During negotiations, ABC said, "We won't agree to this deal unless you agree to the language we have proposed in Paragraph 13. This is a deal-breaker." XYZ reluctantly agreed to include the language proposed in ABC's Paragraph 13. When the final agreement was typed up, though, the typist inadvertently left out that language and both sides signed, not realizing it was not there. After the agreement was signed, ABC discovered the problem and asked XYZ to include the language in the agreement. XYZ refused to include it, citing the parol evidence rule. Is the admission of this evidence barred by the parol evidence rule? A. Yes, because it would supplement the terms of an agreement that was complete and exclusive. B. No, because it is offered for the purpose of modifying the agreement. C. No, because it evidences a mistake in formation. D. No, because it evidences duress on the part of ABC.

The evidence is offered on a formation issue. ABC is attempting to show that the agreement the parties signed is not the one they agreed to and should be reformed to conform to their agreement. This is an issue of mistake, called mistake in integration, and courts do allow the evidence to come in for this purpose. The correct response is C.

During negotiations between two friends over the sale of a boat, the seller tells the buyer that the engine will be warranted to be free of defects for a period of two years. The parties finally sign a written agreement that contains no warranty terms. After 14 months, the engine develops problems because of a defect. A court finds that the writing is a partial integration and that the promise was actually made. Is the seller liable under the warranty? A. Yes, because the gap fillers will supply an implied warranty. B. Yes, because the oral promise does not contradict the terms of the writing. C. No, because the writing was a final expression of their agreement as to the warranty. D. No, because the buyer would have a claim even under the written warranty.

The parties had completed their negotiations and signed a written agreement. Such an agreement is considered final as to the terms that are incorporated therein. Because the agreement is only a partial integration, however, the writing is not complete and there may be other terms that are not found in the writing. In that event, the terms are admissible if they do not contradict the writing. Here, the alleged oral term does not contradict the writing, so it is admissible under the parol evidence rule. That only means that as a matter of law it cannot be excluded as evidence. The party who offered the evidence still needs to prove as a matter of fact that the statement was made, but you are told that the court did find that the promise was actually made. Therefore, the oral term supplies the warranty and under that term, the seller is liable. The correct response is B.

A doughnut manufacturer contracts with a bakery to deliver "ten dozen" assorted doughnuts to the bakery each day. The contract contains a merger clause and is fully integrated. On the first day, the manufacturer delivers 120 doughnuts. The bakery claims that the manufacturer should have delivered 130 doughnuts because there are 13 doughnuts in a baker's dozen. If the matter went to court, would the evidence that there are 13 doughnuts in a baker's dozen be admissible? A. Yes, because evidence of trade usage is admissible to explain the meaning of a term in an agreement, even if it is fully integrated. B. Yes, because evidence of course of dealing is admissible to explain the meaning of a term in an agreement, even if it is fully integrated. C. No, because parol evidence is not admissible to supplement the terms of a fully integrated agreement. D. No, because the parties expressly contracted around the admissibility of trade usage evidence.

The evidence that there are 13 doughnuts in a baker's dozen is trade usage. Therefore, response B is incorrect. It is offered to prove the meaning of the term "ten dozen" in the contract, so this is an issue of interpretation. According to § 2-202(a), such evidence is admissible to explain the meaning of a term in the agreement even if the contract is fully integrated. Unlike other kinds of evidence of meaning, evidence of course of performance, course of dealing, and trade usage are admissible without any initial proof that the term is ambiguous as a matter of law. Official Comment 1(c) provides: 1. This section definitely rejects: . . . (c) The requirement that a condition precedent to the admissibility of the type of evidence specified in paragraph (a) is an original determination by the court that the language used is ambiguous. Therefore, response C is incorrect. As suggested by response D, the parties may be free to contract around this rule, and prohibit evidence of course of performance, course of dealing, and trade usage, but it would be unwise for them to do so. Official Comment 2 explains: 2. Paragraph (a) makes admissible evidence of course of dealing, usage of trade and course of performance to explain or supplement the terms of any writing stating the agreement of the parties in order that the true understanding of the parties as to the agreement may be reached. Such writings are to be read on the assumption that the course of prior dealings between the parties and the usages of trade were taken for granted when the document was phrased. Unless carefully negated they have become an element of the meaning of the words used. Similarly, the course of actual performance by the parties is considered the best indication of what they intended the writing to mean. In any event, there is no evidence that they did so here, so the correct response is A.

A seller sells a machine to a buyer. The contract provides that the seller will service it "every week" for three years. After delivery, the seller finds out that in the trade, these machines are only serviced monthly, so the seller insists that it only has to service the machine monthly. Is the seller right? A. Yes, because the trade usage is monthly. B. Yes, because the course of dealing is monthly. C. No, because the express term is weekly. D. No, because the course of performance is weekly.

The important facts are that the express term of the agreement is "every week" and the trade usage is monthly. In the Code hierarchy of interpretation, express terms govern over trade usage. Therefore, the express term, "every week," governs. The correct response is C.

A widget seller negotiates with a buyer for the purchase of 100 widgets for $10,000, with delivery 30 days from the signing of the agreement. After the parties sign the agreement, which contains a merger clause, the buyer asks the seller if delivery could be in 20 days and the seller says, "Yes, I promise we will do that." The seller does not deliver until 30 days from signing. The buyer sues the seller and the seller seeks to bar the evidence that the seller promised delivery in 20 days. Will the evidence be excluded under the parol evidence rule? A. Yes, because it contradicts a term of the written agreement. B. Yes, because it supplements a written agreement that is complete and exclusive. C. No, because it is offered for the purpose of interpreting the writing. D. No, because it is offered on an issue of modification.

The key here is the fact that "[a]fter the parties sign the agreement . . . the buyer asks the seller. . . ." When the issue is parol evidence, there is always an understanding that is "prior or contemporaneous" to the signing of the agreement. Here, the understanding came after the signing of the agreement. Therefore it is not a parol evidence rule issue but a modification issue. The correct response is D.

A buyer of goods sends a purchase order to a seller on its form. The boilerplate on the form states that the seller is liable for consequential damages. The seller responds with an acknowledgment form that contains a boilerplate that states that the seller is not liable for consequential damages. In addition, the seller's form states, "Acceptance is expressly made conditional on assent to the additional or different terms in this acknowledgment." After the forms are exchanged, the seller ships the goods and the buyer pays for them. The buyer then suffers consequential damages because of a breach by the seller. Is the seller liable for consequential damages? A. No, because there is no contract between the parties. B. No, because the seller's form governs. C. Yes, because the buyer's form governs. D. Yes, because the Code provides for consequential damages in § 2-714(3).

The language employed by the seller satisfies the requirement of § 2- 207(1) after the comma ("unless acceptance is expressly made conditional on assent to the additional or different terms"). That language prevents seller's form from being an acceptance. Under § 2-207(3), however, there has been a contract formed by conduct. In that event, because the parties' writings did not agree on the consequential damages term, both parties' terms on that issue are knocked out and the consequential damages term is supplied by the Code. Therefore, the correct response is D.

During negotiations, the seller of a boat promises the buyer that the engine will be warranted to be free of defects for a period of two years. The parties then sign a written agreement that contains a warranty term with standard disclaimers of express and implied warranties, followed by language stating that "seller warrants that the engine will be free of defects for a period of one year." After 14 months, the engine develops problems because of a defect. A court finds that the writing is not the complete and exclusive statement of the terms of the agreement. Is the seller liable under the warranty? A. Yes, because the parties did not intend their writing to be final as to the warranty. B. Yes, because the oral promise does not contradict the terms of the writing. C. No, because the writing was a final expression of their agreement as to the warranty. D. No, because the buyer would have a claim even under the written warranty.

The parties had completed their negotiations and signed a written agreement. Such an agreement is considered final as to the terms that are incorporated therein. Here, that would include the one-year warranty term. The buyer is offering evidence of an oral understanding that contradicts a term in the writing. This evidence is excluded under the parol evidence rule. Note that it does not matter whether, as a matter of fact, the promise was actually made, for it is excluded as a matter of law. Therefore, the term in the writing supplies the warranty and under that term, the seller is not liable. The correct response is C.

A merchant buyer calls a seller with whom she has not previously dealt and orders 10,000 widgets. The parties discuss the description of the goods, the price, and the delivery terms. The seller puts that information on a sales confirmation and sends it to the buyer. The buyer checks that the information is correct. When the goods arrive, the buyer claims that they are defective, but the seller disputes the claim. The buyer sues the seller, but the seller asks the court to dismiss the suit because the confirmation has an arbitration clause, requiring that the parties bring all claims to arbitration. Is the arbitration clause part of the parties' agreement? A. Yes, because the agreement between the parties is found in the seller's confirmation. B. Yes, because the seller's confirmation proposes arbitration as an additional term and between merchants such terms become part of the contract. C. No, because the agreement between the parties is found in their oral conversation. D. No, because arbitration materially alters the term agreed to in the oral conversation.

The parties reached an oral agreement that was then confirmed by the seller's confirmation. According to § 2-207(2), the additional term (arbitration) in the confirmation is a proposal for addition to the contract. Between merchants, the proposal becomes part of the contract unless it materially alters it. The default rule is that parties have a right to go to court and cannot be compelled to arbitrate unless they agree to arbitration, so that rule is implied as part of the oral agreement. If arbitration is materially different from use of the court system, then the seller's proposed term materially alters the terms of the oral agreement and is not part of the contract. The correct response is D. Response C is not correct because, although the oral agreement is a starting point, it would be supplemented by additional terms in the confirmation that do not materially alter it.

A brickyard entered into a contract to deliver a load of bricks for $5,000. Just before delivery, the buyer tells the seller, "I sure was lucky to get that order of bricks in when I did. There is suddenly a shortage of bricks on the market and if I didn't have them I'd be way behind on my building project." The seller responds, "Since you need them so bad, the price has just gone up to $6,000." Having no reasonable alternative, the buyer agrees to pay the higher price. Is the agreement to pay a $6,000 price enforceable? A. Yes, because the modification was freely made. B. Yes, but the buyer has a remedy for any losses that he suffered. C. No, because there is no consideration for the additional $1,000. D. No, because the modification was not entered in good faith.

The seller has clearly engaged in opportunistic behavior. There is no good reason for the demanded increase other than to take advantage of the buyer's situation. Many modern courts would regard this as an example of economic duress. But even if it is not duress on the part of the seller, the behavior demonstrates a lack of good faith. The correct response is D. The modification is not enforceable, so the price remains $5,000.

A seller promises delivery of 100 type-X widgets by June 1. On May 25, the seller delivers 100 type-Y widgets and the buyer rejects them. The seller says that he has the type-X widgets in stock and promises to deliver them by June 1. The buyer says, "Don't bother to deliver them. You have proven to me that you are an unreliable supplier and I don't want anything more to do with you." The seller does not deliver the widgets. Who is in breach? A. The seller because he did not deliver the proper widgets on May 25. B. The seller because he never delivered the widgets. C. The buyer because he wrongfully rejected the widgets on May 25. D. The buyer because he wrongfully rejected the seller's offer to cure.

There is no doubt that the buyer had a right to reject the widgets on May 25 as this was a material breach. However, the seller had a right to cure under § 2-508(1) because the time for performance had not yet expired. The buyer wrongfully repudiated the contract when it should have given the seller an opportunity to cure. The best response is D.

In a contract for the sale of goods, a merchant offeror's form states, "This contract is governed by the law of Texas." The merchant offeree's form states, "This contract is governed by the law of Vermont." In a jurisdiction that employs the knockout rule, which jurisdiction supplies the governing law? A. Texas, because the offeror should be able to choose the applicable law. B. Texas, because Vermont is materially different. C. Vermont, because it is not materially different from Texas. D. Whichever state prevails when the Code choice of law rules are applied.

These are different terms, and under the knockout rule, both are out and the term supplied by the Code is in. The choice of law rule supplied by the Code is found in § 1-301. Under that rule, if the parties have not agreed on the governing law, "this Act applies to transactions bearing an appropriate relation to this state." Application of that rule would determine which state prevails. The correct response is D.

A seller entered into an oral agreement with a buyer for the sale of a widget for $10,000. Shortly thereafter, the seller sent the buyer a written confirmation, confirming the sale of a widget from the seller to the buyer for $10,000. The buyer received the confirmation and did not object to it. At the time for performance, the seller tendered the widget and the buyer refused to accept it. What is the buyer's best defense to a claim that it is in breach for refusing to accept the widget? A. There is no writing signed by the buyer. B. The confirmation was not sent within a reasonable time. C. The confirmation was not sufficient against the sender. D. Both parties are not merchants.

These facts invoke the confirmation exception of § 2-201(2). Response A is true, but it is not the best response because the point of the subsection is to provide an exception to the rule that a writing signed by the party against whom enforcement is sought is required to make a contract within the Statute of Frauds enforceable. If the writing is sufficient against the sender, then it is sufficient against the receiving party. The other responses indicate elements that have to be satisfied to come within the confirmation exception. There are facts indicating that B and C are satisfied, for the facts state that the confirmation was sent "shortly thereafter," which is probably within a reasonable time, and that the confirmation stated that a contract for sale had been made between the parties, which would be sufficient against the seller. Other terms of the agreement can be supplied. There are no facts, however, indicating that both parties are merchants, so D is the best response.

A buyer purchases a machine "AS IS" for $1,000. When the machine arrives, the buyer inspects it and, finding no problems, accepts it. A week later, the machine fails to work and its value to the buyer is substantially impaired. Can the buyer revoke acceptance? A. Yes, because the nonconformity substantially impairs its value to the buyer, and the buyer's acceptance was reasonably induced by the difficulty of discovery before acceptance. B. Yes, because there is material breach by the seller. C. No, because once there is acceptance, it is too late to revoke. D. No, because there was no nonconformity.

This is a bit of a trick question. The key fact is that the machine was purchased "AS IS." According to § 2-316(3)(a), which we explored in Chapter 10.C, this language effectively disclaims the warranty of merchantability. Because the seller made no promise that the machine functions, the machine as delivered conforms to the contract. The correct response is D. If the facts were otherwise, and the seller had promised that the machine would function, then the correct response would be A. In those circumstances, revocation would be appropriate because a latent defect could not be discovered on inspection.

A department store runs an ad in a local newspaper that states, "Black lapin stoles. $100. This sale price is effective for the next 10 days." The department store then decides to stop this sale and the next day it runs an ad in the newspaper that states, "Effective at the close of business today, black lapin stoles are no longer available at the sale price. We regret any inconvenience this may cause." Is the sale price for the stoles still effective the next day? A. Yes, because it was a merchant's firm offer that is not revocable during the time stated. B. Yes, because the method of revocation through the newspaper was not effective. C. No, because the newspaper advertisement was not an offer. D. No, because the offer was not a firm offer because it was made to the general public, not to merchants.

This is a bit tricky — you might even say it was a trick question. When we work through the elements of § 2-205 to see if they are satisfied, the first element is that there is "an offer." You may recall from Contracts (and I gave you a hint by using a hypothetical involving the sale of black lapin stoles, the subject matter of the case of Lefkowitz v. Great Minneapolis Surplus Store, 86 N.W.2d 689 (Minn. 1957)) that an advertisement is generally not an offer, and this one does not have the earmarks of an offer. Therefore, if it isn't an offer, it cannot be a firm offer, so the correct response is C. Note that response B is not correct because in general an offer can be effectively revoked in a medium similar to the medium in which it was made. Response D is not correct because, although a firm offer can only be made by a merchant, it does not have to be made to a merchant.

A seller sells 100 type-A widgets to a buyer and agrees to arrange for shipping to the buyer at the buyer's expense. The seller notifies the buyer that the goods have been shipped. While the goods are en route, they are hijacked and never found. As between the seller and the buyer, who is responsible for the loss? A. The seller, because this was a destination contract. B. The seller, because it was negligent in choosing the carrier. C. The buyer, because the risk of loss passed when the goods were delivered to the carrier. D. The buyer, because the shipment was at the buyer's expense.

This is a shipment contract. Notice that the mere fact that the seller has arranged for shipment to the buyer does not make it a destination contract. Obviously, every shipping contract has to have a destination. In a shipment contract, under § 2-509(1)(a), "the risk of loss passes to the buyer when the goods are duly delivered to the carrier." The correct response is C.

A buyer and a seller entered into a written agreement for the delivery of a truckload of bricks in 30 days. The contract contained a NOM clause. Shortly after the agreement was made, the buyer asked the seller if it could deliver in 20 days and the seller responded that that would not be a problem. The buyer then hired a crew of bricklayers to begin work in 20 days. The next day the seller told the buyer that he changed his mind, and he insisted on the 30-day delivery date as specified in the written agreement. Is the oral modification to 20 days effective? A. Yes, because NOM clauses are not enforceable. B. Yes, because the buyer relied on the modification. C. No, because the NOM clause is enforceable. D. No, because the seller retracted his waiver within a reasonable time.

This is a typical situation. Even though the agreement contained an enforceable NOM clause, the parties waived their right to insist on it when they made an oral modification. The seller could retract his waiver, but not after the buyer had relied on it. The best response is B. Sometimes a contract includes not only a NOM clause, but a "no waiver" clause providing that waivers are not effective unless written. Courts will generally disregard this provision, finding that parties are free to change their minds and that the no waiver clause was waived along with the NOM clause.

A consumer is test driving a car at a used car dealership. She hears a sound under the car and asks the salesperson about it. The salesperson says, "Don't worry about that. We stand behind the cars we sell. If anything goes wrong in the next 60 days, we will fix it free of charge." Relieved, the consumer agrees to buy the car. Two days later, the transmission fails. She brings it back to the dealer and tells the manager what the salesperson said. The manager tells her that the salespeople aren't allowed to negotiate terms and all the terms are found in the writing. He points out that the writing effectively disclaims all warranties and conspicuously states that the car is sold "AS IS." He also points out a merger clause that states that there are no promises or understandings other than those found in the writing. Is the consumer likely to recover? A. Yes, because the evidence does not contradict the writing. B. Yes, because the parol evidence rule does not apply to consumers. C. No, because the writing is a partial integration and the evidence does not contradict the writing. D. No, because the writing is a full integration.

This is a very common sad story. A court must first determine whether the agreement is fully integrated in the writing. The writing contains a merger clause and extensive language concerning warranty, so in context it appears to be complete and exclusive. The best response is D. It is true that a court might find that the boilerplate language is not determinative on the issue of integration.

A seller sells 100 type-A widgets to a buyer and agrees to arrange for shipping to the buyer at its own (the seller's) expense. The seller notifies the buyer that the goods have been shipped. The seller inadvertently loaded 100 type-B widgets. While the goods are en route, they are hijacked and never found. As between the seller and the buyer, who is responsible for the loss? A. The seller, because its agreement to pay for the shipping made this a destination contract. B. The seller, because the goods did not conform to the contract. C. The buyer, because the risk of loss passed when the goods were delivered to the carrier. D. The buyer, because it had a reasonable opportunity to obtain insurance after it was notified of the shipment.

This is still a shipment contract. The fact that the seller has agreed to pay for shipping means just that, and does not alter the risk of loss. As a general rule, in a shipment contract, under § 2-509(1)(a) "the risk of loss passes to the buyer when the goods are duly delivered to the carrier." However, the general rule is subject to an exception under § 2-510(1): "Where a tender or delivery of goods so fails to conform to the contract as to give a right of rejection the risk of their loss remains on the seller until cure or acceptance." Here, the seller shipped goods that did not conform to the contract. Therefore, the risk of loss remained on the seller during shipment. The correct response is B.

A contractor sends a company its bid on a construction job. The bid consists of specifications and prices, followed by a number of boilerplate provisions. The company responds by writing up the price and specifications on its own form, which it sends to the contractor. The contractor completes the project and the company inspects it after 25 days. The company finds some unfinished work and demands payment for it. The contractor claims that he is not responsible for the unfinished work because the form he sent says that inspection must be completed within 20 days after construction, and the company did not timely inspect. However, the company's form indicates that it has 30 days after completion to inspect. Who is responsible for the unfinished work? The owner, because the contractor's form governs. The contractor, because the company's form governs. Neither, because there is no agreement on this term. Both terms are knocked out and the party who would be responsible is supplied by trade usage or by acourt.

This problem involves construction services rather than the sale of goods. Therefore, the common law rather than Article 2 applies. Under the common law mirror-image rule, if the acceptance does not match the offer, then it is a counteroffer. Here, the contractor's bid was an offer, and the company's response with different terms was a counteroffer. The contractor accepted those terms by conduct when it completed the project. Although I am not completely confident in this outcome, I think most courts would conclude that the correct response is B.

An American importer contracted to buy 100 cases of wine from a German seller. The buyer agreed to pay for shipping. The seller arranged for shipment by ship from the port of Hamburg to the port of Wilmington, North Carolina, where the buyer was to receive it, but the seller did not notify the buyer. The ship sank en route and the buyer never received the wine. Which party is responsible for the risk of loss? A. The seller, because this was a destination contract. B. The seller, because even though this was a shipment contract, the seller failed to notify the buyer of the shipment. C. The buyer, because this was a shipment contract. D. The buyer, because it agreed to pay for shipping.

This was a shipment contract, because the seller agreed only to deliver the goods to the port and not to the buyer. In a shipment contract, according to § 2-509, "the risk of loss passes to the buyer when the goods are duly delivered to the carrier." Normally, therefore, the risk of loss would be on the buyer in this situation. However, the seller did not notify the buyer as required by § 2-504(c). Although failure to give notice is not material in all circumstances, under these facts, a court held that the risk of loss was on the seller. It stated, "The requirement of prompt notification by the seller, as used in [§ 2-504(c)], must be construed as taking into consideration the need of a buyer to be informed of the shipment in sufficient time for him to take action to protect himself from the risk of damage to or loss of the goods while in transit." See Rheinberg-Kellerei GMBH v. Vineyard Wine Co., Inc., 281 S.E.2d 425 (N.C. App. 1981). The best response is B.

On a Friday, a customer who rarely purchased art walked into an art gallery and saw a painting displayed that he liked. He said to the manager, "I'll give you $5,000 for that painting." The manager said, "It's a deal." The customer then explained that he would return with the money on Monday and the manager said, "That won't be a problem. I'll hold the painting for you in the back." The customer then said, "Not that I don't trust you, but I'm going to confirm our deal so there is some evidence of it." He wrote on a piece of paper, "John Smith agreed to purchase the Picasso painting La Reve from the Gilderson Gallery on March 1." He signed the paper and handed it to the manager. When the customer returned on Monday with the money, the manager apologetically explained that the painting had inadvertently been sold to someone else. The customer insisted that they had an enforceable contract. Did they? A. No, because the confirmation was not sufficient. B. No, because both parties were not merchants. C. Yes, because there was a writing signed by the party against whom enforcement is sought. D. Yes, because there is a confirmation that was sufficient.

Under subsection (1), there is not a writing signed by the art gallery, which is the party against whom enforcement is sought. Is the gallery bound under the confirmation exception? The confirmation here sufficiently described the transaction. The price was missing, but that can be supplied by evidence of the commercial setting in which the transaction was made. The problem is that the purchaser is not a merchant. Therefore, subsection (2) is not satisfied. The best response is B.

A buyer of goods sends a purchase order to a seller on its form. The seller responds with an acknowledgment form that has additional and different terms. In addition, the seller's form states, "Acceptance is expressly made conditional on assent to the additional or different terms in this acknowledgment." When it receives the form, the buyer refuses to go through with the deal. Is the buyer in breach? A. No, because under the common law there was a counteroffer that was not acted on. B. No, because the Code says there was no acceptance, and therefore no contract. C. Yes, because the Code says there is an acceptance and therefore a contract. D. Yes, because there was acceptance by conduct.

Under the Code, the language employed by the seller satisfies the requirement of § 2-207(1) after the comma ("unless acceptance is expressly made conditional on assent to the additional or different terms"). That language prevents seller's form from being an acceptance. Because there was no acceptance of buyer's offer, buyer cannot be in breach of contract. The correct response is B.

A contract that contains a NOM clause allows the seller 20 days to deliver the goods. The seller calls the buyer and asks if he may instead have 30 days and the buyer agrees. Immediately after agreeing, the buyer calls the seller back and says, "I changed my mind. That modification is not enforceable because it is not in writing." Is the modification enforceable? A. No, because there is a NOM clause. B. No, because the buyer retracted his waiver before the seller could rely on it. C. Yes, because the buyer waived his right to insist on the NOM clause by making an oral modification. D. Yes, because under the Code an NOM clause is not effective.

Under the rule of § 2-209(4), the buyer has waived the right to insist that the oral modification is not enforceable because of the NOM clause. But if the buyer reaches the seller before the seller has taken action in reliance on the oral agreement, then pursuant to subsection (5), the buyer can retract the waiver. The correct response is B.

A consumer wishing to purchase a computer goes to the seller's website to place an order. The buyer fills in the blanks that describe the goods and the quantity. The buyer is given an opportunity to read the terms and conditions, but without reading them, he clicks the button labeled "I accept." What are the terms of the contract that has been formed? A. The terms in seller's terms and conditions. B. The terms in the buyer's counteroffer. C. We would have to perform an analysis of the additional and different terms under § 2-207 to answer the question. D. Where the parties' terms don't agree, the terms are the default terms of the UCC.

Under these facts, the buyer has agreed to the terms offered by the seller, so the best response is A. Because the buyer does not have an opportunity to bargain for terms, this transaction represents a modern contract of adhesion. The buyer has no opportunity to negotiate for terms prior to entering the contract. After formation, if the buyer objects to a term, it may argue that the term is unconscionable or contrary to the buyer's reasonable expectations. Note that this is becoming the prevailing method by which goods are bought and sold, so the issue raised in Question 14 may soon be moot.

A coal-fired power station became short of fuel and sent a fax to a number of coal companies that said, "We are offering to buy 100 tons of coal that contains at least 80% carbon at market price." The Carbon County Coal Company replied by e-mail, "We accept your offer. The coal will be shipped tomorrow." Carbon County only had coal that contained 75% carbon, so when it shipped the coal it notified the buyer that it was sending the coal as an accommodation. Is Carbon County in breach? A. Yes, because a contract was formed when it accepted the offer and it then shipped nonconforming goods. B. Yes, because a contract was formed when it shipped nonconforming goods. C. No, because its reply by e-mail did not constitute an acceptance. D. No, because it offered the nonconforming goods as an accommodation.

When an offer is made by fax, e-mail seems to be a reasonable medium of acceptance, so C is not a correct response. The solution to this problem is to determine whether the acceptance was by promise or performance. Although the offer invited either method of acceptance, when Carbon County sent its e-mail, it appears that the language of the e-mail constituted an acceptance by promise. Therefore, B is not a correct response because there was acceptance by promise rather than by performance. Even though Carbon County notified the buyer that it was shipping nonconforming goods as an accommodation, this fact is irrelevant because it had already accepted by promise. The best response is A. Had the acceptance been by performance, D would have been the correct response.

A business has ordered a new delivery truck and puts its old truck up for sale. The parties agree on all the terms of the sale, but the seller says, "We need this truck until our new one comes, so we won't give you delivery until then." The buyer says, "No problem." They both sign a writing that contains the complete and exclusive statement of the terms of their agreement. The buyer then demands immediate delivery of the truck. The seller reminds the buyer of their oral understanding and the buyer says, "Ha Ha! I don't see that in the contract." Is the evidence of the oral understanding admissible? A. No, because the writing is a complete and exclusive statement of the terms of their agreement. B. No, because reasonable parties would have included this understanding in the writing. C. Yes, because the evidence shows that performance is subject to an oral condition. D. Yes, because the evidence shows a defense to contract formation

You may agree with response B. That response is subsumed in response A, however, because B is merely evidence of A. Nevertheless, in this case the parol evidence is offered for the purpose of proving a condition, and most courts will admit evidence of a condition even if the writing is a complete and exclusive statement of the terms of the parties' agreement. The correct response is C. Note that UCC § 2-202 does not address this issue. Under § 1- 103(b), however, the Code is supplemented by the common law, and this is a common law rule, as stated in Restatement § 217.

Statute reader. Hard up for money, Joe takes his laptop computer to a pawnshop. In the transaction with the pawnshop, Joe is loaned a certain amount of money and agrees that the laptop is to be security for the loan. Before the date on which Joe would be in default, Jan buys the laptop from the pawnshop. Did the pawnshop have the power to transfer title to Jan? Hint: Review the definition of buyer in ordinary course of business in § 1-201(b)(9). A. Yes, because she is a good faith purchaser for value. B. Yes, because she is a buyer in ordinary course of business. C. No, because she should have known that the sale violates the rights of another person in the goods. D. No, because she is not a buyer in ordinary course of business.

You would think the answer here would be B, because it is the ordinary course of a pawnshop's business to sell goods that once belonged to others. But if you carefully read the definition of "buyer in ordinary course of business" in § 1-201(b)(9), you noted that it refers to "a person that buys goods in good faith, without knowledge that the sale violates the rights of another person in the goods, and in the ordinary course from a person, other than a pawnbroker, in the business of selling goods of that kind." I can't explain the pawnbroker exception, other than to point out the obvious distinction between the inventory of a pawnshop and the inventory of other businesses to which one might entrust goods. Because she purchased from a pawnshop, Jan is not a buyer in ordinary course of business. The best response is D.


संबंधित स्टडी सेट्स

unit 6 genetic expression & regulation

View Set

Economics Chapter 1 and 2 Test Review

View Set

APUSH Vol. 1 to 1877 Ch. 18 Renewing the Sectional Struggle, 1848-1854

View Set

433 test 3 musculoskeletal review questions

View Set

Chap 3 3.3 a membrane seperates each cell from its surrounding

View Set

Anatomical Features (Markings) of Bones

View Set